Set of Double Cosets












1














Consider a group $G$ with subgroups $H$ and $K$. The double cosets ${ HgK : g in G }$ partition $G$. It seems this is just a set, not a group action. But I thought I might be missing something- can this be viewed as a group action of some group? I suppose it is the set of $K$-orbits of the group action of $G$ on $G/H$ (and vice versa). Also, I thought perhaps that $G$ could act, where for $g, g_0 in G$, $g cdot Hg_0 K = H g g_0 K$, but it seems this is not well defined.



More generally, I would appreciate any answer stating whether there is a universal property of this construction in some category.










share|cite|improve this question





























    1














    Consider a group $G$ with subgroups $H$ and $K$. The double cosets ${ HgK : g in G }$ partition $G$. It seems this is just a set, not a group action. But I thought I might be missing something- can this be viewed as a group action of some group? I suppose it is the set of $K$-orbits of the group action of $G$ on $G/H$ (and vice versa). Also, I thought perhaps that $G$ could act, where for $g, g_0 in G$, $g cdot Hg_0 K = H g g_0 K$, but it seems this is not well defined.



    More generally, I would appreciate any answer stating whether there is a universal property of this construction in some category.










    share|cite|improve this question



























      1












      1








      1







      Consider a group $G$ with subgroups $H$ and $K$. The double cosets ${ HgK : g in G }$ partition $G$. It seems this is just a set, not a group action. But I thought I might be missing something- can this be viewed as a group action of some group? I suppose it is the set of $K$-orbits of the group action of $G$ on $G/H$ (and vice versa). Also, I thought perhaps that $G$ could act, where for $g, g_0 in G$, $g cdot Hg_0 K = H g g_0 K$, but it seems this is not well defined.



      More generally, I would appreciate any answer stating whether there is a universal property of this construction in some category.










      share|cite|improve this question















      Consider a group $G$ with subgroups $H$ and $K$. The double cosets ${ HgK : g in G }$ partition $G$. It seems this is just a set, not a group action. But I thought I might be missing something- can this be viewed as a group action of some group? I suppose it is the set of $K$-orbits of the group action of $G$ on $G/H$ (and vice versa). Also, I thought perhaps that $G$ could act, where for $g, g_0 in G$, $g cdot Hg_0 K = H g g_0 K$, but it seems this is not well defined.



      More generally, I would appreciate any answer stating whether there is a universal property of this construction in some category.







      group-theory group-actions






      share|cite|improve this question















      share|cite|improve this question













      share|cite|improve this question




      share|cite|improve this question








      edited Nov 25 at 15:46

























      asked Nov 25 at 15:30









      Dean Young

      1,464720




      1,464720






















          1 Answer
          1






          active

          oldest

          votes


















          1














          If I understand your first question correctly, then the answer is that these double cosets are the orbits of the (right) action of the direct product $H times K$ on $G$ given by
          $$
          g^{(h, k)} = h^{-1} g k.
          $$

          (I denote the action by an exponent.)






          share|cite|improve this answer





















          • Or the left action given by $(h,k)g:=hgk^{-1}$.
            – anon
            Dec 3 at 2:01











          Your Answer





          StackExchange.ifUsing("editor", function () {
          return StackExchange.using("mathjaxEditing", function () {
          StackExchange.MarkdownEditor.creationCallbacks.add(function (editor, postfix) {
          StackExchange.mathjaxEditing.prepareWmdForMathJax(editor, postfix, [["$", "$"], ["\\(","\\)"]]);
          });
          });
          }, "mathjax-editing");

          StackExchange.ready(function() {
          var channelOptions = {
          tags: "".split(" "),
          id: "69"
          };
          initTagRenderer("".split(" "), "".split(" "), channelOptions);

          StackExchange.using("externalEditor", function() {
          // Have to fire editor after snippets, if snippets enabled
          if (StackExchange.settings.snippets.snippetsEnabled) {
          StackExchange.using("snippets", function() {
          createEditor();
          });
          }
          else {
          createEditor();
          }
          });

          function createEditor() {
          StackExchange.prepareEditor({
          heartbeatType: 'answer',
          autoActivateHeartbeat: false,
          convertImagesToLinks: true,
          noModals: true,
          showLowRepImageUploadWarning: true,
          reputationToPostImages: 10,
          bindNavPrevention: true,
          postfix: "",
          imageUploader: {
          brandingHtml: "Powered by u003ca class="icon-imgur-white" href="https://imgur.com/"u003eu003c/au003e",
          contentPolicyHtml: "User contributions licensed under u003ca href="https://creativecommons.org/licenses/by-sa/3.0/"u003ecc by-sa 3.0 with attribution requiredu003c/au003e u003ca href="https://stackoverflow.com/legal/content-policy"u003e(content policy)u003c/au003e",
          allowUrls: true
          },
          noCode: true, onDemand: true,
          discardSelector: ".discard-answer"
          ,immediatelyShowMarkdownHelp:true
          });


          }
          });














          draft saved

          draft discarded


















          StackExchange.ready(
          function () {
          StackExchange.openid.initPostLogin('.new-post-login', 'https%3a%2f%2fmath.stackexchange.com%2fquestions%2f3012979%2fset-of-double-cosets%23new-answer', 'question_page');
          }
          );

          Post as a guest















          Required, but never shown

























          1 Answer
          1






          active

          oldest

          votes








          1 Answer
          1






          active

          oldest

          votes









          active

          oldest

          votes






          active

          oldest

          votes









          1














          If I understand your first question correctly, then the answer is that these double cosets are the orbits of the (right) action of the direct product $H times K$ on $G$ given by
          $$
          g^{(h, k)} = h^{-1} g k.
          $$

          (I denote the action by an exponent.)






          share|cite|improve this answer





















          • Or the left action given by $(h,k)g:=hgk^{-1}$.
            – anon
            Dec 3 at 2:01
















          1














          If I understand your first question correctly, then the answer is that these double cosets are the orbits of the (right) action of the direct product $H times K$ on $G$ given by
          $$
          g^{(h, k)} = h^{-1} g k.
          $$

          (I denote the action by an exponent.)






          share|cite|improve this answer





















          • Or the left action given by $(h,k)g:=hgk^{-1}$.
            – anon
            Dec 3 at 2:01














          1












          1








          1






          If I understand your first question correctly, then the answer is that these double cosets are the orbits of the (right) action of the direct product $H times K$ on $G$ given by
          $$
          g^{(h, k)} = h^{-1} g k.
          $$

          (I denote the action by an exponent.)






          share|cite|improve this answer












          If I understand your first question correctly, then the answer is that these double cosets are the orbits of the (right) action of the direct product $H times K$ on $G$ given by
          $$
          g^{(h, k)} = h^{-1} g k.
          $$

          (I denote the action by an exponent.)







          share|cite|improve this answer












          share|cite|improve this answer



          share|cite|improve this answer










          answered Nov 25 at 16:23









          Andreas Caranti

          55.9k34295




          55.9k34295












          • Or the left action given by $(h,k)g:=hgk^{-1}$.
            – anon
            Dec 3 at 2:01


















          • Or the left action given by $(h,k)g:=hgk^{-1}$.
            – anon
            Dec 3 at 2:01
















          Or the left action given by $(h,k)g:=hgk^{-1}$.
          – anon
          Dec 3 at 2:01




          Or the left action given by $(h,k)g:=hgk^{-1}$.
          – anon
          Dec 3 at 2:01


















          draft saved

          draft discarded




















































          Thanks for contributing an answer to Mathematics Stack Exchange!


          • Please be sure to answer the question. Provide details and share your research!

          But avoid



          • Asking for help, clarification, or responding to other answers.

          • Making statements based on opinion; back them up with references or personal experience.


          Use MathJax to format equations. MathJax reference.


          To learn more, see our tips on writing great answers.





          Some of your past answers have not been well-received, and you're in danger of being blocked from answering.


          Please pay close attention to the following guidance:


          • Please be sure to answer the question. Provide details and share your research!

          But avoid



          • Asking for help, clarification, or responding to other answers.

          • Making statements based on opinion; back them up with references or personal experience.


          To learn more, see our tips on writing great answers.




          draft saved


          draft discarded














          StackExchange.ready(
          function () {
          StackExchange.openid.initPostLogin('.new-post-login', 'https%3a%2f%2fmath.stackexchange.com%2fquestions%2f3012979%2fset-of-double-cosets%23new-answer', 'question_page');
          }
          );

          Post as a guest















          Required, but never shown





















































          Required, but never shown














          Required, but never shown












          Required, but never shown







          Required, but never shown

































          Required, but never shown














          Required, but never shown












          Required, but never shown







          Required, but never shown







          Popular posts from this blog

          How do I know what Microsoft account the skydrive app is syncing to?

          When does type information flow backwards in C++?

          Grease: Live!